Jump to content

le_hoang1995's Content

There have been 342 items by le_hoang1995 (Search limited from 09-06-2020)



Sort by                Order  

#333640 Ảnh thành viên

Posted by le_hoang1995 on 09-07-2012 - 16:28 in Góc giao lưu

Cứu tôi với ... !!! .... !
Máu be bét ra rồi !

Posted Image

Cần đưa ngay đến bệnh viện
___________________
P/s: Mình là người trong bức ảnh !

Có thấy tí máu nào đâu, kiểu này trông giống như Bạch Tuyết ăn trái táo độc bị tắc cổ hơn



#332802 Ảnh thành viên

Posted by le_hoang1995 on 07-07-2012 - 12:56 in Góc giao lưu

Chỗ này sao trông giống cái ảnh của Linh, còn đây là ai thì mình không biết



#535392 Topic về Bất đẳng thức, cực trị THCS

Posted by le_hoang1995 on 29-11-2014 - 21:08 in Bất đẳng thức và cực trị

Nhờ các bạn giúp mình bài này:
Cho các số dương x, y, z thoả mãn x + y + z = 1. Tìm MIN của P = ax2  + by2 + cz2 (với a, b, c >0).

Áp dụng BĐT Cauchy-Schwarz ta được

$(ax^2+by^2+cz^2)\left ( \frac{1}{a}+\frac{1}{b}+\frac{1}{c} \right )\geq (x+y+z)^2=1$
$\Rightarrow ax^2+by^2+cz^2\geq \frac{abc}{ab+bc+ca}$

Đẳng thức xảy ra khi và chỉ khi $x+y+z=1$ và $ax=by=cz$

Đặt $ax=by=cz=t$, thay vào $x+y+z=1$, ta được

$t.\left ( \frac{1}{a}+\frac{1}{b}+\frac{1}{c} \right )=1$
$\Rightarrow t=\frac{abc}{ab+bc+ca}$
$\Rightarrow \left\{\begin{matrix}
x=\frac{bc}{ab+bc+ca}\\ y=\frac{ca}{ab+bc+ca}
\\z=\frac{ab}{ab+bc+ca}

\end{matrix}\right.$




#324398 Topic bất đẳng thức THCS (2)

Posted by le_hoang1995 on 12-06-2012 - 14:52 in Bất đẳng thức và cực trị

Bài 374: Cho các số thực dương a,b,c thoả mãn a+b+c=3. Chứng minh rằng:
$\frac{a^{3}}{b^{2}+c}+\frac{b^{3}}{c^{2}+a}+\frac{c^{3}}{a^{2}+b}\geqslant \frac{3}{2}$

Theo BĐT Cauchy-schwarz, ta có
$$VT\geq \frac{(a^2+b^2+c^2)^2}{ab^2+bc^2+ca^2+ab+bc+ca}$$
Ta có: $$3(a^2+b^2+c^2)=(a+b+c)(a^2+b^2+c^2)=\sum a^3+\sum a^2b+\sum ab^2$$
Theo BĐT Am-GM thì $a^3+ac^2\geq 2a^2c$.

Tương tự rồi cộng lại, ta suy ra $a^3+b^3+c^3+a^2b+b^2c+ac^2\geq 2(b^2a+c^2b+a^2c)$
$$\Rightarrow \sum a^3+\sum a^2b+\sum ab^2\geq 3(b^2a+c^2b+a^2c)\Leftrightarrow 3(a^2+b^2+c^2)\geq 3(b^2a+c^2b+a^2c)$$
$$\Leftrightarrow a^2+b^2+c^2\geq b^2a+c^2b+a^2c$$
Dế thấy $a^2+b^2+c^2\geq ab+bc+ca$
$$\Rightarrow VT\geq \frac{(a^2+b^2+c^2)^2}{b^2a+c^2b+a^2c+ab+bc+ca}\geq \frac{(a^2+b^2+c^2)^2}{2(a^2+b^2+c^2)}=\frac{(a^2+b^2+c^2)}{2}\geq \frac{3}{2}$$



#313274 Topic bất đẳng thức THCS (2)

Posted by le_hoang1995 on 29-04-2012 - 10:59 in Bất đẳng thức và cực trị

334.
Cho a,b,c>0 tm $a^{4}+b^{4}+c^{4}=3$.cmr
$\sum \frac{a^{2}}{b+c}\geq \frac{3}{2}$


Theo BĐT chê-bư-sép ta có $\sum \frac{a^2}{b+c}\geq \frac{1}{3}.(a^4+b^4+c^4)\left [\frac{1}{a^2(b+c)}+\frac{1}{b^2(a+c)} +\frac{1}{c^2(b+a)} \right ]\geq \frac{9}{a^2(b+c)+b^2(c+a)+c^2(a+b)}$

Ta dễ chứng minh $a^2(b+c)+b^2(c+a)+c^2(a+b)\leq 2(a^3+b^3+c^3)$ theo AM-GM.

Và theo Holder ta có $3.(a^4+b^4+c^4)(a^4+b^4+c^4)(a^4+b^4+c^4)\geq (a^3+b^3+c^3)^4$

$\Rightarrow a^3+b^3+c^3\leq 3$

$\Rightarrow a^2(b+c)+b^2(c+a)+c^2(a+b)\leq 2(a^3+b^3+c^3)\leq 2.3=6$.

Thay vào trên, ta suy ra ĐPCM



#325002 Topic bất đẳng thức THCS (2)

Posted by le_hoang1995 on 14-06-2012 - 09:08 in Bất đẳng thức và cực trị

Bài 382: Cho $a,b,c>0$. Chứng minh rằng: $$\sqrt{\frac{a^2+b^2+c^2}{3}}\le\frac{1}{3}(\frac{ab}{c}+\frac{bc}{a}+\frac{ac}{b})$$

Sử dụng BĐT cơ bản $(x+y+z)^2\geq 3(xy+yz+zx)$
$$\Rightarrow \left ( \frac{ab}{c}+\frac{bc}{a}+\frac{ca}{b} \right )^2\geq 3\left ( b^2+c^2+a^2 \right )$$
$$\Leftrightarrow \frac{1}{3}\left ( \frac{ab}{c}+\frac{bc}{a}+\frac{ca}{b} \right )\geq \sqrt{\frac{a^2+b^2+c^2}{3}}$$



#300253 Topic bất đẳng thức THCS (2)

Posted by le_hoang1995 on 21-02-2012 - 00:05 in Bất đẳng thức và cực trị

Em post thêm một số bài để duy trì topic này :D
Bài 280: Cho $a,b,c>0$ thỏa mãn ${a^2} + {b^2} + {c^2} = 3$
CMR: $\left( {\frac{4}{{{a^2} + {b^2}}} + 1} \right)\left( {\frac{4}{{{b^2} + {c^2}}} + 1} \right)\left( {\frac{4}{{{c^2} + {a^2}}} + 1} \right) \ge 3{\left( {a + b + c} \right)^2}$


Bài này mình làm như sau.

Theo Holder, ta có

$\left( {\frac{4}{{{a^2} + {b^2}}} + 1} \right)\left( {\frac{4}{{{b^2} + {c^2}}} + 1} \right)\left( {\frac{4}{{{c^2} + {a^2}}} + 1} \right)\ge \left [\frac{4}{\sqrt[3]{(a^2+b^2)(b^2+c^2)(c^2+a^2)}}+1 \right ]^3$

$\ge\left [ \frac{4}{ \frac{2(a^2+b^2+c^2)}{3}}+1 \right ]^3=3^3$

$VP=3(a+b+c)^2\leq 3*(a^2+b^2+c^2)*3=3^3$

Suy ra ĐPCM



#332329 Topic bất đẳng thức THCS (2)

Posted by le_hoang1995 on 05-07-2012 - 22:14 in Bất đẳng thức và cực trị

Thêm một bài duy trì topic.
Bài 415. Cho a,b,c dương, chứng minh
$$\sqrt{\frac{2a}{b+c}}+\sqrt{\frac{2b}{c+a}}+\sqrt{\frac{2c}{a+b}}\geq \frac{(a+b+c)^2}{a^2+b^2+c^2}$$



#336673 Topic bất đẳng thức THCS (2)

Posted by le_hoang1995 on 17-07-2012 - 00:08 in Bất đẳng thức và cực trị

Cho $a,b,c> 0$
Chứng minh: $\frac{a^{3}}{2b+3c}+\frac{b^{3}}{2c+3a}+\frac{c^{3}}{2a+3b}\geq \frac{a^{2}+b^{2}+c^{2}}{5}$

Sử dụng BĐT cơ bản $a^2+b^2+c^2 \geq ab+bc+ca$.
Áp dụng BĐT AM-GM ta được
$$VT\geq \frac{(a^2+b^2+c^2)^2}{2ab+3ac+2bc+3ab+2ac+3bc}=\frac{(a^2+b^2+c^2)^2}{5(aB+bc+ca)}\geq \frac{a^2+b^2+c^2}{5}$$
ĐPCM
___________________________________________________________

Bài 458 Cho các số thực không âm a,b,c thỏa mãn $min(a+b;b+c;c+a)>0$, và $a^2+b^2+c^2 =2(ab+bc+ca)$. Tìm giá trị nhỏ nhất của $$A=\sqrt{\frac{ab}{a^2+b^2}}+\sqrt{\frac{bc}{b^2+c^2}}+\sqrt{\frac{ca}{c^2+a^2}}$$

Bài 459 . Cho các số thực dương thỏa mãn $\sqrt{a}+\sqrt{b}+\sqrt{c}=1$. Tìm giá trị lớn nhất của
$$A=\sqrt{\frac{ab}{a+b+2c}}+\sqrt{\frac{bc}{b+c+2a}}+\sqrt{\frac{ca}{c+a+2b}}$$
Bài 459 cũ có trong topic rồi nên em tạm del :)



#298829 Topic bất đẳng thức THCS (2)

Posted by le_hoang1995 on 10-02-2012 - 17:37 in Bất đẳng thức và cực trị

Chỗ mình tô đỏ bạn có nhầm không. Theo mình:
\[\frac{{{a^2}}}{{a + 2{b^3}}} = a - \frac{{2a{b^3}}}{{a + 2{b^3}}} = a - \frac{{2a{b^3}}}{{a + {b^3} + {b^3}}} \ge a - \frac{{2a{b^3}}}{{3{b^2}\sqrt[3]{a}}}\]


CAO XUÂN HUY nói đúng rồi

Cuối cùng ta cần chứng minh $\sum \sqrt[3]{(ab)^2}\leq 3\Leftrightarrow \frac{(a+b+c)^4}{3}\geq (a+b+c)(b+c+a)(ab+bc+ca)\geq (\sqrt[3]{(ab)^2}+\sqrt[3]{(bc)^2}+\sqrt[3]{(ca)^2})^3\Rightarrow$ đúng theo holder



#339324 Topic bất đẳng thức THCS (2)

Posted by le_hoang1995 on 23-07-2012 - 20:03 in Bất đẳng thức và cực trị

Mình làm bài này nếu đề là $\frac{3}{2\sqrt[3]{(abc)^2}}$ :D
Áp dụng bất đẳng thức $Cauchy-Schwarz$ dạng $Engel$ và $AM-GM$ ta có:
$\frac{a}{bc(b+c)}+\frac{b}{ac(a+c)}+\frac{c}{ab(a+b)}=\frac{a^2}{abc(b+c)}+\frac{b^2}{abc(a+c)}+\frac{c^2}{abc(a+b)}\geq \frac{(a+b+c)^2}{2abc(a+b+c)}=\frac{a+b+c}{2abc}\geq \frac{3}{2\sqrt[3]{(abc)^2}}$

Bổ sung thêm một cách nữa nhé, dạng của nó khiến ta nghĩ đến BĐT Chebusep.

Không mất tổng quát, giả sử $ a \ge b \ge c$, khi đó cách bộ sau đơn điệu cùng chiều

$\left ( \frac{a}{b+c};\frac{b}{c+a};\frac{c}{a+b} \right )$ và $\left ( \frac{1}{bc};\frac{1}{ca};\frac{1}{ab} \right )$

Theo BĐT Chebusep và Nesbit
$$\frac{a}{bc(b+c)}+\frac{b}{ac(a+c)}+\frac{c}{ab(a+b)}\geq \frac{1}{3}.\left ( \frac{a}{b+c}+\frac{b}{c+a}+\frac{c}{a+b} \right ).\left ( \frac{1}{bc}+\frac{1}{ca}+\frac{1}{ab} \right )\geq \frac{1}{3}.\frac{3}{2}.\frac{3}{\sqrt[3]{(abc)^2}}=\frac{3}{2\sqrt[3]{(abc)^2}}$$



#327497 Topic bất đẳng thức THCS (2)

Posted by le_hoang1995 on 21-06-2012 - 09:21 in Bất đẳng thức và cực trị

Bài 390 Chứng minh rằng với $a,b,c>0$ ta có
$$\frac{a+b}{c}+\frac{b+c}{a}+\frac{c+a}{b}\geq 3+\frac{(a^2+b^2+c^2)(ab+bc+ca)}{abc(a+b+c)}$$



#297937 Topic bất đẳng thức THCS (2)

Posted by le_hoang1995 on 03-02-2012 - 19:29 in Bất đẳng thức và cực trị

Tặng bài khác vậy. :D
Bài 238. Cho các số thực dương $a,b,c$ thỏa mãn
$$\frac{1}{a+1}+\frac{1}{b+1}+\frac{1}{c+1}=2$$
Chứng minh $$\frac{1}{4a+1}+\frac{1}{4b+1}+\frac{1}{4c+1}\ge 1$$


Đặt $\frac{1}{a+1}=x,...\Rightarrow a=\frac{1-x}{x},...$ và $\frac{1}{4a+1}=\frac{x}{4-3x}$

Vậy ta có $x+y+z=2$ và $3(x^2+y^2+z^2)\geq (x+y+z)^2=4$

$VT=\sum \frac{x}{4-3x}\geq \frac{(x+y+z)^2}{4(x+y+z)-3(x^2+y^2+z^2)}\geq \frac{2^2}{4*2-4}=1$

Dấu bằng khi $x=y=z=\frac{2}{3}\Leftrightarrow a=b=c=\frac{1}{2}$



#295431 Topic bất đẳng thức THCS (2)

Posted by le_hoang1995 on 22-01-2012 - 20:38 in Bất đẳng thức và cực trị

Một số bài tương đối nhẹ, làm xong thì nghỉ ngơi mà đón giao thừa nhé !
Bài 157. Cho $x, y, z$ là các số thực dương thoả mãn điều kiện $x^2 + y^2 + z^2 = 1$ Chứng minh rằng :
$$x^2yz + y^2xz + z^2xy \le \dfrac{1}{3}$$


Thêm một cách nữa, sử dụng chê-bư-sép
Không mất tính tổng quát giả sử $x\geq y\geq z$
$\Rightarrow x^2\geq y^2\geq z^2;xy\geq xz\geq yz$
$VT\leq \frac{1}{3}(x^2+y^2+z^2)(yz+xz+xy)\leq \frac{1}{3}(x^2+y^2+z^2)^2=\frac{1}{3}$



#295436 Topic bất đẳng thức THCS (2)

Posted by le_hoang1995 on 22-01-2012 - 20:55 in Bất đẳng thức và cực trị

Chà, sôi nổi quá ! Có vẻ như mấy bài này nhẹ quá thì phải !mình tiếp tục nè (cũng dễ nhìn)
bài 161. Cho $0<a, b, c<1$. Chứng minh rằng :
$$\dfrac{a}{1 - a} + \dfrac{b}{1 - b} + \dfrac{c}{1 - c} \ge \dfrac{3\sqrt[3]{abc}}{1 - \sqrt[3]{abc}}$$


Sử dụng chê-bư-sép và bunhia và cosi

giả sử $a\geq b\geq c\Rightarrow \frac{1}{1-a}\geq \frac{1}{1-b}\geq \frac{1}{1-c}$

$VT\geq \frac{1}{3}(a+b+c)(\frac{1}{1-a}+\frac{1}{1-b}+\frac{1}{1-c})\geq \frac{1}{3}*3\sqrt[3]{abc}*(\frac{9}{3-(a+b+c)})\geq VP$



#295427 Topic bất đẳng thức THCS (2)

Posted by le_hoang1995 on 22-01-2012 - 20:24 in Bất đẳng thức và cực trị

Chúc cách thành viên VMF 1 năm mới may mắn, hạnh phúc nha.


Bài 158. cho $a, b, c$ là các số thực thoả mãn điều kiện $a + b + c = 1$ .Chứng minh rằng:
$$a^2 + b^2 + c^2 + 1 \ge 4(ab + bc + ca)$$ (1)


$(1)\Leftrightarrow a^2+b^2+c^2+1\geq 2(a+b+c)^2-2(a^2+b^2+c^2)$

$\Leftrightarrow 3(a^2+b^2+c^2)+1\geq 2$

$\Leftrightarrow 3(a^2+b^2+c^2)\geq 1$ Đúng theo Bunhia.



#295110 Topic bất đẳng thức THCS (2)

Posted by le_hoang1995 on 21-01-2012 - 22:07 in Bất đẳng thức và cực trị

Bài 144. Một bài cũng khá dễ thở.
Cho a, b, c, d là các sổ thực dương thoả mãn điều kiện$a + b + c + d = 1$. Chứng minh rằng :
$$6(a^3 + b^3 + c^3 + d^3) \ge a^2 + b^2 + c^2 + d^2 + \dfrac{1}{8}.$$


Mình xin làm bài này, sử dụng Holder và chê-bư-sép.

$(a^{3}+b^{3}+c^{3}+d^{3})(1+1+1+1)(1+1+1+1)\geq (a+b+c+d)^{3}=1$

$\Leftrightarrow 2(a^{3}+b^{3}+c^{3}+d^{3})\geq \frac{1}{8} (1)$

$(a^{3}+b^{3}+c^{3}+d^{3})\geq \frac{1}{4}(a^2+b^2+c^2+d^2)(a+b+c+d)$

$\Leftrightarrow 4(a^{3}+b^{3}+c^{3}+d^{3})\geq a^2+b^2+c^2+d^2(2)$

Cộng theo vế 1 và 2, ta có ĐPCM.

Dấu bằng xảy ra khi $a=b=c=d=1/4$



#295038 Topic bất đẳng thức THCS (2)

Posted by le_hoang1995 on 21-01-2012 - 14:55 in Bất đẳng thức và cực trị

Bài 130: Tìm $min$ của biểu thức:
$\frac{a^{6}}{b^{3}+c^{3}}+\frac{b^{6}}{a^{3}+c^{3}}+\frac{c^{6}}{b^{3}+a^{3}}$.
Trong đó $a,b,c$ là các số thực dương thỏa mãn điều kiện $a+b+c=1$
zzZZzzZZzzZZzzZZzzZZzzZZzzZZzzZZzzZZ.zZmanZz


Một cách nữa cho bài này, sử dụng chê-bư-sép
$\frac{a^{6}}{b^{3}+c^{3}}+\frac{b^{6}}{a^{3}+c^{3}}+\frac{c^{6}}{b^{3}+a^{3}}\geq \frac{1}{3}(a^{3}+b^{3}+c^{3})(\frac{a^{3}}{b^3+c^3}+\frac{b^{3}}{c^{3}+a^{3}}+\frac{c^{3}}{a^{3}+b^{3}})$

$\geq \frac{1}{3}*\frac{(a+b+c)^{3}}{9}*\frac{3}{2}= \frac{1}{18}$



#295610 Topic bất đẳng thức THCS (2)

Posted by le_hoang1995 on 23-01-2012 - 20:38 in Bất đẳng thức và cực trị

Bài 168

Đặt $x=\frac{a^2}{b^2},y=\frac{a^2}{c^2} \Rightarrow x+y=a^2(\frac{1}{b^2}+\frac{1}{c^2})\geq \frac{4a^2}{b^2+c^2}\geq \frac{4a^2}{a^2}=4$

$P=x+y+\frac{1}{x}+\frac{1}{y}\geq \frac{x+y}{4}+\frac{4}{x+y}+\frac{3(x+y)}{4}\geq 2+\frac{3*4}{4}=5$

Dấu bằng khi $x=y=2$ tương đương $a^2=2b^2=2c^2$

$\Rightarrow P_{min}=5$ khi và chỉ khi $a=\sqrt{2}b=\sqrt{2}c$



#296297 Topic bất đẳng thức THCS (2)

Posted by le_hoang1995 on 25-01-2012 - 12:55 in Bất đẳng thức và cực trị

186. Cho $a, b, c$ là các số thực dương :$a + b + c = 1$. Chứng minh rằng:
$$\dfrac{a^3}{(1 - a)^2} + \dfrac{b^3}{(1 - b)^3} + \dfrac{c^3}{(1 - c)^3} \ge \dfrac{1}{4}$$


Bài này có 1 cách nữa là dùng chê-bư-sép và BĐT Bunhia

$$VT=\sum \frac{a^3}{(b+c)^2}\geq \frac{1}{3}(a+b+c)\left [(\frac{a}{b+c})^2+(\frac{b}{c+a})^2+(\frac{c}{a+b})^2 \right ]\geq \frac{1}{3}*\frac{(\frac{a}{b+c}+\frac{b}{c+a}+\frac{c}{a+b})^2}{3}\geq \frac{1}{4}$$

Dấu bằng xảy ra khi $a=b=c=1/3$

__________________________________

Hoặc dùng Côsi

$\sum ( \frac{a^3}{(b+c)^2}+\frac{a}{4} )\geq \sum \frac{a^2}{b+c} \Leftrightarrow VT+\frac{1}{4}\geq \sum \frac{a^2}{b+c} (1)$

Mà $\sum (\frac{a^2}{b+c}+\frac{b+c}{4})\geq a\Leftrightarrow \sum \frac{a^2}{b+c}\geq \frac{a+b+c}{2}=\frac{1}{2} (2)$

Từ (1) và (2) suy ra ĐPCM



#297826 Topic bất đẳng thức THCS (2)

Posted by le_hoang1995 on 02-02-2012 - 20:37 in Bất đẳng thức và cực trị

Bài 234.
Tìm $a, b$ để
$P = a^4 - a^3b + a^2b^2 - ab^3 - 18ab + b^4 + 2005$ Đạt giá trị nhỏ nhất
Bài 235. Cho $x, y$ là các số thực thoả mãn : $x^2 + y^2 - 4x - 6y + 12 = 0$
Tìm $x, y$ sao cho $A = x^2 + y^2 $ đạt giá trị lớn nhất.


Mình xin làm hai bài này.

Bài 234 bằng phân tích trực tiếp, ta có

$P=(a^3-b^3)(a-b)+(ab-9)^2-81+2005\geq 1924$

vậy $P_{min}=1924 \Leftrightarrow a=b=_{-}^{+}3$

Bài 235 điều kiện tương đương $(x-2)^2+(y-3)^2=1$

Đặt $x=2+a, y=3+b$ suy ra $a^2+b^2=1$

$A=(2+a)^2+(3+b)^2=a^2+b^2+13+2(2a+3b)\leq 1+13+2*\sqrt{(4+9)(a^2+b^2)}$

$=14+2\sqrt{13}$

Giải hệ dấu bằng khi $x=2+\frac{2}{\sqrt{13}},y=3+\frac{3}{\sqrt{13}}$



#297317 Topic bất đẳng thức THCS (2)

Posted by le_hoang1995 on 29-01-2012 - 20:12 in Bất đẳng thức và cực trị

Bài 223.
Cho $a, b, c > 0$ . Chứng minh bất đẳng thức
$$\sqrt[3]{(\dfrac{a}{b + c})^2} + \sqrt[3]{(\dfrac{b}{c + a})^2} + \sqrt[3]{(\dfrac{c}{a + b})^2} \ge \dfrac{3\sqrt[3]{2}}{2}$$


Bài này mình làm sử dụng côsi

$\sqrt[3]{(\frac{a}{b+c})^2}=\sqrt[3]{\frac{2a^3}{2a(b+c)^2}}\geq \sqrt[3]{\frac{2a}{(\frac{2(a+b+c)}{3})^3}}=\frac{3\sqrt{2}a}{2(a+b+c)}$

Tương tự 2 BĐT nữa rồi cộng lại ta có ĐPCM



#296366 Topic bất đẳng thức THCS (2)

Posted by le_hoang1995 on 25-01-2012 - 18:18 in Bất đẳng thức và cực trị

Bài 194: Cho x,y,z thực dương. CMR:
$\sqrt{(x+y)(x+z)(y+z)}(\sqrt{x+y}+\sqrt{x+z}+\sqrt{z+x})\geq 4(xy+yx+xz)$
Chuyên Vĩnh Phúc 2008-2009


mình chứng minh kiểu này hơi phức tạp một chút

Ta có: $\left ( a+b\right )\left (b+c \right )\left ( c+a \right )= \left ( ab+bc+ca \right )\left ( a+b+c \right )-abc\geq \dfrac{8}{9}\left ( ab+bc+ca \right )\left ( a+b+c \right )$

$\Leftrightarrow \dfrac{\left ( a+b \right )\left ( b+c \right )\left ( c+a \right )}{8}\geq \dfrac{\left ( ab+bc+ca\right )\left ( a+b+c \right )}{9}$ (1)

Vì $\left ( a+b+c \right )^{2}\geq 3\left ( ab+bc+ca \right )$ nên

$\dfrac{\left ( ab+bc+ca\right )\left ( a+b+c \right )}{9}\geq \sqrt{(\dfrac{ab+bc+ca}{3})^{3}}$ (2)


Từ 1 và 2 suy ra $\sqrt[3]{\dfrac{\left ( a+b \right )\left ( b+c \right )\left ( c+a \right )}{8}}\geq \sqrt{\dfrac{ab+bc+ca}{3}}$
__________________________________________________________________


Để dễ theo dõi, đặt
$ A=\left ( a+b \right )\left ( b+c \right )\left ( c+a \right ), B=ab+bc+ca$

Ta có:$ \sqrt[3]{\frac{A}{8}}\geq \sqrt{\frac{B}{3}}$

Suy ra $A\geq 8\sqrt{(\frac{B}{3})^3}$ suy ra $A^2\geq 8^2*(\frac{B}{3}^3)$

__________________________________________________________________

$VT=\sqrt{(a+b)(b+c)(c+a)}(\sqrt{a+b}+\sqrt{b+c}+\sqrt{c+a})\geq \sqrt{A}*3*\sqrt[3]{\sqrt{A}}=3\sqrt[3]{A^2}$

$\geq 3\sqrt[3]{8^2*(\frac{B}{3})^3}=4B$

ĐPCM

Làm xong bài này vất quá :icon6:



#296357 Topic bất đẳng thức THCS (2)

Posted by le_hoang1995 on 25-01-2012 - 17:53 in Bất đẳng thức và cực trị

Bài 193: Cho $a<b<c<d$ là các hằng số cho trước. Với giá trị nào của x thì biểu thức sau nhận giá trị nhỏ nhất
$$f(x)=|x-a|+|x-b|+|x-c|+|x-d|\geq |2x-a-b|+|2x-c-d|=|2x-a-b|+|c+d-2x|\geq |c+d-a-b|$$
ĐTTS vào lớp 10 trường PTNK ĐHQG TPHCM 1994-1995


Sử dụng BĐT $|a|+|b| \geq |a+b|$ dấu bằng khi $ab\geq0$

$f(x)=|x-a|+|x-b|+|x-c|+|x-d|\geq |2x-a-b|+|2x-c-d|=|2x-a-b|+|c+d-2x|$
$\geq |c+d-a-b|$

Dấu bằng xảy ra khi $(x-a)(x-b)\geq 0$
và $(x-c)(x-d)\geq 0$
và $(2x-a-b)(2x-c-d)\leq 0$

tương đương $b< x <c$



#295031 Topic bất đẳng thức THCS (2)

Posted by le_hoang1995 on 21-01-2012 - 14:44 in Bất đẳng thức và cực trị

Quy nạp theo cách này mình thấy vẫn chưa ổn lắm. Không biết ý kiến các bạn như thế nào ?

Theo mình thì B2 của bạn mới chỉ giả sử BĐT đúng với k số, chứng minh BĐT cũng đúng với k+1 số, nhưng bạn đã nhầm lẫn khi dùng chung kí hiệu nên mới suy ra $x_{k+1}=1$.
Nếu mình nhớ không lầm thì đây là bài tập trong SBT toán 11 có thì phải